PT36.S1.Q20 - editorialist: to ensure justice in the legal system

LSATHopeful-2LSATHopeful-2 Alum Member
edited July 2017 in Logical Reasoning 109 karma

Hey Everyone,

So I'm currently drilling NA question types through the Cambridge packet things. I'm looking at question 20 in section 1 of PT 36, and it says its an NA question type.

I got the right answer quite quickly, but for the life of me I can't seem to figure out how this isn't also a sufficient answer choice - something which has never happened to me before. What I mean by that is, answer choice E being true seems to be sufficient to make the argument true.

Core:

P1: Ensuring Justice in the legal system ---> Citizens capable of criticizing anyone involved in determining punishments
P2: Legal system's purpose is to deter ---> System falls into hands of experts whose specialty is to assess how potential lawbreakers are affected by the system's punishments
P3: Most citizens lack knowledge about such matters
C: Justice is therefore not ensured in the legal system

E) Citizens without knowledge about how the legal system's punishments affect potential lawbreakers are incapable of criticizing experts in that area

I JUST THOUGHT OF THIS: Is the reason why E isn't sufficient for the argument is because P1 never states the number of citizens who must be capable of criticizing lawmakers? P3 says MOST citizens, meaning some citizens do possess the knowledge necessary to criticize lawmakers, and therefore justice CAN be ensured in the legal system? The argument requires it to be necessary, if you didn't need to understand the affect of the legal system's punishments, then the conclusion is completely wrong. But with E being true, the conclusion can still be true - we just don't know if it has to be true.

https://7sage.com/lsat_explanations/lsat-36-section-1-question-20/

Comments

  • OlamHafuchOlamHafuch Alum Member
    2326 karma

    I didn't read through the details about the particular question, but it is indeed possible for a necessary assumption to be sufficient. A necessary assumption does not have to be sufficient, and a sufficient assumption can overshoot and therefore be unnecessary, but you can have an assumption that is both necessary and sufficient.

Sign In or Register to comment.